24. Applications of Taylor Series

c.1. Integrals

There are many integrals which can not be computed exactly. We previously saw how to approximate a definite integral as a Riemann sum. However, if we know a Taylor series for the integrand, we can

Evaluate \(\displaystyle \int e^{-x^2}\,dx\) as a power series centered at \(x=0\). Find its radius of convergence.

Substitute \(x\to-x^2\) into the exponential series \[ e^x=\sum_{n=0}^\infty \dfrac{x^n}{n!} \] to get \[ e^{-x^2}=\sum_{n=0}^\infty (-1)^n\dfrac{x^{2n}}{n!} \] Now integrate both sides: \[ \int e^{-x^2}\,dx =\sum_{n=0}^\infty (-1)^n\dfrac{x^{2n+1}}{(2n+1)n!}+C \] Since the series for \(e^x\) has an infinite radius of convergence, so does the series for \(e^{-x^2}\) and that for \(\displaystyle \int e^{-x^2}\,dx\). Alternatively, you can apply the Ratio Test directly to the series \(\displaystyle \sum_{n=0}^\infty (-1)^n\dfrac{x^{2n+1}}{(2n+1)n!}\) to see it converges for all \(x\).


Approximate \(\displaystyle \int_0^1 e^{-x^2}\,dx\) to within \(\pm 10^{-3}\).

In the previous example, we found a series expansion for \(\displaystyle \int e^{-x^2}\,dx\). So \[ \int_0^1 e^{-x^2}\,dx =\left[\sum_{n=0}^\infty (-1)^n\dfrac{x^{2n+1}}{(2n+1)n!}\right]_0^1 =\sum_{n=0}^\infty \dfrac{(-1)^n}{(2n+1)n!} \] This is an alternating series. So we can approximate it by a finite number of terms with the error bounded by the absolute value of the next term. The term with \(n=5\) is \[ \left|\dfrac{(-1)^5}{(2\cdot5+1)5!}\right| =\dfrac{1}{11\cdot120}=\dfrac{1}{1320} \lt 10^{-3} \] So it is sufficient to keep the terms up to \(n=4\). \[\begin{aligned} \int_0^1 e^{-x^2}\,dx &\approx\dfrac{(-1)^0}{(2\cdot0+1)0!} +\dfrac{(-1)^1}{(2\cdot1+1)1!} +\dfrac{(-1)^2}{(2\cdot2+1)2!} \\ &\qquad+\dfrac{(-1)^3}{(2\cdot3+1)3!} +\dfrac{(-1)^4}{(2\cdot4+1)4!} \pm \dfrac{(-1)^5}{(2\cdot5+1)5!} \\ &=1-\dfrac{1}{3}+\dfrac{1}{10}-\dfrac{1}{42}+\dfrac{1}{216} \pm \dfrac{1}{1320} \approx .7475\pm .0007 \end{aligned}\]

Evaluate \(\displaystyle \int \arctan(x^2)\,dx\) as a power series centered at \(x=0\). Find its radius of convergence.

\[ \arctan x =\sum_{n=0}^\infty (-1)^n\dfrac{x^{2n+1}}{2n+1} =x-\dfrac{x^3}{3}+\dfrac{x^5}{5}+\cdots \]

\(\begin{aligned}\displaystyle \int \arctan(x^2)\,dx &=\sum_{n=0}^\infty (-1)^n\dfrac{x^{4n+3}}{(2n+1)(4n+3)}+C \\ &=\dfrac{x^3}{3}-\dfrac{x^7}{21}+\dfrac{x^{11}}{55}+\cdots+C \end{aligned}\)
\(R=1\)

We substitute \(x\to x^2\) into the series: \[ \arctan x=\sum_{n=0}^\infty (-1)^n\dfrac{x^{2n+1}}{2n+1} =x-\dfrac{x^3}{3}+\dfrac{x^5}{5}+\cdots \] to get: \[ \arctan(x^2)=\sum_{n=0}^\infty (-1)^n\dfrac{x^{4n+2}}{2n+1} =x^2-\dfrac{x^6}{3}+\dfrac{x^{10}}{5}+\cdots \] We integrate both sides: \[\begin{aligned} \int \arctan(x^2)\,dx &=\sum_{n=0}^\infty (-1)^n\dfrac{x^{4n+3}}{(2n+1)(4n+3)}+C \\ &=\dfrac{x^3}{3}-\dfrac{x^7}{21}+\dfrac{x^{11}}{55}+\cdots+C \end{aligned}\] The series for \(\arctan x\) converges for \(|x| \lt 1\). So the series for \(\arctan(x^2)\) converges for \(|x^2| \lt 1\) which is equivalent to \(|x| \lt 1\). So the series for \(\displaystyle \int \arctan(x^2)\,dx\) also converges for \(|x| \lt 1\). Alternatively, the Ratio Test says the series converges when: \[\begin{aligned} \lim_{n\to\infty}\left|\dfrac{a_{n+1}}{a_n}\right| &=\lim_{n\to\infty} \left|\dfrac{x^{4(n+1)+3}}{(2[n+1]+1)(4[n+1]+3)} \cdot\dfrac{(2n+1)(4n+3)}{x^{4n+3}}\right| \\ &=|x^4| \lt 1 \end{aligned}\] or \(|x| \lt 1\). So the radius of convergence is \(R=1\).

Approximate \(\displaystyle \int_0^{0.1} \arctan(x^2)\,dx\) to within \(\pm 10^{-10}\).

\[\begin{aligned} \int \arctan(x^2)\,dx &=\sum_{n=0}^\infty (-1)^n\dfrac{x^{4n+3}}{(2n+1)(4n+3)}+C \\ &=\dfrac{x^3}{3}-\dfrac{x^7}{21}+\dfrac{x^{11}}{55}+\cdots+C \end{aligned}\]

\(\displaystyle \int_0^{0.1} \arctan(x^2)\,dx \approx3.333\,285\,714\times10^{-4}\pm1.8\times10^{-13} \)

\[\begin{aligned} \int \arctan(x^2)\,dx &=\sum_{n=0}^\infty (-1)^n\dfrac{x^{4n+3}}{(2n+1)(4n+3)}+C \\ &=\dfrac{x^3}{3}-\dfrac{x^7}{21}+\dfrac{x^{11}}{55}+\cdots+C \end{aligned}\] So \[\begin{aligned} \int_0^{0.1} \arctan(x^2)\,dx &=\sum_{n=0}^\infty (-1)^n\dfrac{(0.1)^{4n+3}}{(2n+1)(4n+3)} \\ &=\dfrac{(0.1)^3}{3}-\dfrac{(0.1)^7}{21}+\dfrac{(0.1)^{11}}{55}+\cdots \end{aligned}\] This is an alternating series and \(\dfrac{(0.1)^{11}}{55} \lt 10^{-10}\). So \[\begin{aligned} \int_0^{0.1} \arctan(x^2)\,dx &\approx\dfrac{(0.1)^3}{3}-\dfrac{(0.1)^7}{21}\pm \dfrac{(0.1)^{11}}{55} \\ &\approx3.333\,285\,714\times10^{-4}\pm1.8\times10^{-13} \end{aligned}\]

In the above examples and exercises, the series were all alternating. So we could use the alternating series bound to bound the remainder. In the exercises on the next page, the series are not alternating. So we will need to use the Taylor bound on the remainder.

© MYMathApps

Supported in part by NSF Grant #1123255